Example: intersection of compact sets which is NOT compact

Click For Summary
The discussion revolves around finding subsets M_1 and M_2 that are compact in the topology defined by S = {(a,b) : 0 < a < b < 1} Union {R}, but whose intersection is not compact. Participants clarify the meaning of compactness, emphasizing that a set is compact if every cover by open sets has a finite subcover. The conversation confirms that the topology is based on the real line, and participants acknowledge understanding key concepts such as open sets and covers. The focus remains on the challenge of identifying the specific subsets that meet the criteria outlined in the homework statement. Ultimately, the discussion aims to deepen understanding of compactness within this unique topological framework.
SD123
Messages
3
Reaction score
0

Homework Statement


Let S = {(a,b) : 0 < a < b < 1 } Union {R} be a base for a topology. Find subsets M_1 and M_2 which are compact in this topology but whose intersection is not compact.

Homework Equations


The Attempt at a Solution


I'm not even sure what it means for an element of S to be compact, so I haven't been able to make any attempt at a solution.
 
Physics news on Phys.org
I assume you're working on the real line, right?

Do you know the meaning of the terms 'open set', 'cover' and ' finite subcover'?
I don't mean to be condescending; just to know.
 
I assume it is the real line, and so the topological space will be (R,S).

Yes I do know what open set/cover/finite sub cover mean
 
The most general definition is that a subset S is compact iff (def.) every cover of S by open sets has a finite subcover. There are more specialized results, e.g., for R^n, compactness is equivalent to being closed and bounded,and, for metric spaces you have, e.g., every sequence has a convergent subsequence, but the first one covers all cases.
 
Question: A clock's minute hand has length 4 and its hour hand has length 3. What is the distance between the tips at the moment when it is increasing most rapidly?(Putnam Exam Question) Answer: Making assumption that both the hands moves at constant angular velocities, the answer is ## \sqrt{7} .## But don't you think this assumption is somewhat doubtful and wrong?

Similar threads

Replies
12
Views
2K
  • · Replies 1 ·
Replies
1
Views
2K
  • · Replies 14 ·
Replies
14
Views
2K
Replies
1
Views
2K
  • · Replies 5 ·
Replies
5
Views
1K
  • · Replies 5 ·
Replies
5
Views
3K
Replies
10
Views
2K
  • · Replies 1 ·
Replies
1
Views
2K
  • · Replies 11 ·
Replies
11
Views
3K
  • · Replies 3 ·
Replies
3
Views
2K